Đến nội dung

tuan101293 nội dung

Có 316 mục bởi tuan101293 (Tìm giới hạn từ 06-06-2020)



Sắp theo                Sắp xếp  

#245080 help me

Đã gửi bởi tuan101293 on 24-10-2010 - 12:20 trong Bất đẳng thức và cực trị

bài 2
http://www.nxbgd.vn/...mp;ReportID=631



#253256 Dãy số đầu năm

Đã gửi bởi tuan101293 on 14-02-2011 - 22:05 trong Đại số

4,
Ta xét đồ thị $y=\sqrt[3]{px}$ với $1\le x\le (p-1)(p-2)$ ©
Đặt tổng đã cho là S,ta có số điểm nguyên dương nằm dưới đồ thị © chính là tổng S
Quay ngược trục tọa độ(tráo x cho y),ta có đồ thị © chính là $y=\dfrac{x^3}{p}$ với $1\le x\le \sqrt[3]{p(p-1)(p-2)}$
Số điểm nguyên nằm dưới đồ thị © bây h chính là tổng $S'=\sum [\dfrac{k^3}{p}]$ (chạy từ $1\le x\le [\sqrt[3]{p(p-1)(p-2)}]=p-2$)
nên ta có
$S+S'=(p-1)(p-2)^2$ (số điểm nguyên trong hình chữ nhật)
mà $S'=\dfrac{(p-1)(p-2)(p+1)}{4}-[\dfrac{(p-1)^3)}{p}]$
(tính S' đơn giản vì $[\dfrac{k^3}{p}]+[\dfrac{(p-k)^3}{p}]=\dfrac{k^3}{p}+\dfrac{(p-k)^3}{p}-1$)
nên ta có ngay ĐPCM
...
hy vọng được ăn mỳ tôm :)



#253263 Dãy số đầu năm

Đã gửi bởi tuan101293 on 14-02-2011 - 22:37 trong Đại số

Em có thấy cách này trên mathlinks của pco.
cach 2 bai 4



#253062 Số học.

Đã gửi bởi tuan101293 on 10-02-2011 - 15:43 trong Số học

1)CMR:
với mọi số nguyên tố lẻ p đều ko tồn tại các số nguyên dương m,n thỏa mãn:;
$\dfrac{1}{p} = \dfrac{1}{m^2} + \dfrac{1}{n^2}$

quy đồng ta có
$p(m^2+n^2)=m^2n^2$
WLOG, giả sử $p|m$
suy ra $p^2|m^2n^2|p(m^2+n^2)$
suy ra $p|m^2+n^2$ suy ra $p|n$
đặt $m=pm_1,n=pn_1$
suy ra $\dfrac{1}{m_1^2}+\dfrac{1}{n_1^2}=p>1$ vô lý
ĐPCM



#245042 My Inequality 2

Đã gửi bởi tuan101293 on 24-10-2010 - 10:07 trong Bất đẳng thức - Cực trị

Đặt $a=\dfrac{1}{x}$,....
thì
$\sum_{cyc}a^3b^2=\sum_{cyc}\dfrac{1}{x^3y^2}=\dfrac{\sum_{cyc}yz^3}{x^3y^3z^3}\le \dfrac{(\sum a^2)}{3abc}$ (bdt vasc)
nên ta sẽ CM
$(\sum a^2)(\sum ab)abc(\sum a^2b^2)^2\le 81$
đến đây hy vọng là ra
:leq



#244202 My Inequality 2

Đã gửi bởi tuan101293 on 18-10-2010 - 21:02 trong Bất đẳng thức - Cực trị

Định xài $(\sum xy)(\sum_{cyc} x^2y)\le 9$ mà ko ra
Chú post lời giải lên đi.:geq



#454632 nghịch lý monty hall

Đã gửi bởi tuan101293 on 02-10-2013 - 14:13 trong IQ và Toán thông minh

cái này là conditional probability rõ ràng, sự kiện sau liên quan đến sự kiện trước, bạn rai_2601 nói không đúng nhé.

Cm thì có 1 bạn ở trên nói nghe 33+33=66 cũng đúng  nhưng để lập luận toán học thì người ta thường dùng Bayes theorem. có 1 dòng thôi à.

trước mình học cái này cũng hơi bất ngờ. Nhuwng hài ở chỗ là ai cũng nghĩ là 50% thành ra chơi ô cửa bí mật chả ai đổi =)).




#229426 Cũng thú vị đây!

Đã gửi bởi tuan101293 on 18-02-2010 - 13:02 trong Bất đẳng thức - Cực trị

http://www.mathlinks...pic.php?t=86713
Đây là lời giải của arqady bên mathlinks,dài nhưng đúng
:D



#229428 Giúp em với

Đã gửi bởi tuan101293 on 18-02-2010 - 13:08 trong Phương trình hàm

thay y=0 suy ra f(1)=1
thay x=1 suy ra f(1+y)+f(1-y)-2f(1+y)=2y(3y-1)
thay y=-y suy ra f(1+y)-f(1-y)=2y(3y+1)
cộng 2 cái lại suy ra 0=6y^2 suy ra y=0
suy ra SAI ĐỀ
:D



#220917 1/a^2+2bc

Đã gửi bởi tuan101293 on 20-11-2009 - 08:32 trong Bất đẳng thức - Cực trị

Tớ gõ các này mục đích là tập tành cái chuyện soạn văn bản bằng latex thôi :(

Tớ lần đầu gõ latex nên mò mẫm có hơi lâu :leq ( độ .... 4 -5 tiếng gì đấy :leq ) , lệnh thì tớ biết rất ít ---> trình bày có hơi khó coi 1 xíu mong mọi người bỏ qua cho :luoi .

Cái lời giải này của VIMF tớ cũng chưa check nữa ... anh em xem có thấy cái gì sai thì nói tớ sửa nhá :luoi

Thì mình cũng bảo là 27 thì có lời giải đẹp mà:leq
còn cái 310 thì nhìn kinh quá,ko dám expand.
thks
p/s:lời giải quy đồng của mình đẹp hơn:D



#220840 1/a^2+2bc

Đã gửi bởi tuan101293 on 19-11-2009 - 16:25 trong Bất đẳng thức - Cực trị

Thay đổi một chút nhá!!!! Hãy cm:
$\dfrac{1}{2a^2+bc}+\dfrac{1}{2b^2+ca}+\dfrac{1}{2c^2+ab} \ge \dfrac{ab+bc+ca}{a^2b^2+b^2c^2+c^2a^2}$
:(

Công nhận là thế thật:
ta có $a^2b^2+b^2c^2+c^2a^2\ge 2bca^2+b^2c^2=bc(2a^2+bc)$
tương tự ghép vào ta có đpcm



#253406 bd thức

Đã gửi bởi tuan101293 on 18-02-2011 - 18:38 trong Bất đẳng thức - Cực trị

chú ý
$\dfrac{a^2}{b}+b-2a=\dfrac{(a-b)^2}{b}$
suy ra $VT=\sum_{cyc}\dfrac{a^2}{b}-a-b-c=\dfrac{(a-b)^2}{b}+\dfrac{(b-c)^2}{c}+\dfrac{(c-a)^2}{a}\ge \dfrac{(a-b)^2}{b}+\dfrac{(b-a)^2}{c+a}\ge (b-a)^2(\dfrac{4}{a+b+c})=VP$
ĐPCM



#220443 1/a^2+2bc

Đã gửi bởi tuan101293 on 15-11-2009 - 18:15 trong Bất đẳng thức - Cực trị

Với cùng điệu kiện trên thì bất đẳng thức sau vẫn đúng :

$\dfrac{1}{{{a^2} + 2bc}} + \dfrac{1}{{{b^2} + 2ca}} + \dfrac{1}{{{c^2} + 2ab}} \ge \dfrac{{ab + bc + ca}}{{{a^2}{b^2} + {b^2}{c^2} + {c^2}{a^2}}} + \dfrac{{37}}{2}.\dfrac{{{{\left[ {\left( {a - b} \right)\left( {b - c} \right)\left( {c - a} \right)} \right]}^2}}}{{{{\left( {a + b + c} \right)}^6}\left( {ab + bc + ca} \right)}}$

Mình nghĩ là bdt này có vấn đề
chắc là phải thay 37 thành 27 và 1 số cái nữa



#230022 Giúp em với

Đã gửi bởi tuan101293 on 24-02-2010 - 09:31 trong Phương trình hàm

Bài sau thì quen rồi :D
$\dfrac{f(x)}{x^2}+1=f(\dfrac{1}{x})+1=f(\dfrac{x+1}{x})=\dfrac{f(\dfrac{x}{x+1})}{(\dfrac{x}{x+1})^2}=\dfrac{f(1-\dfrac{1}{x+1})}{(\dfrac{x}{x+1})^2}=\dfrac{1-f(\dfrac{1}{x+1})}{(\dfrac{x}{x+1})^2}=\dfrac{1-\dfrac{f(x+1)}{(x+1)^2}}{(\dfrac{x}{x+1})^2}=\dfrac{(x+1)^2-f(x+1)}{x^2}$
suy ra f(x)=x với mọi x khác 0
cho x=0 vào cái đầu suy ra f(0)=0
f(x)=x với mọi x



#253500 bd thức

Đã gửi bởi tuan101293 on 20-02-2011 - 10:20 trong Bất đẳng thức - Cực trị

Thế này nhé
xài cái kia thì suy ra ta sẽ CM
$\sum \dfrac{4a(b+c)(ab+bc+ca)}{(b+c)^2(a+b+c)^2}\ge 2$
tương đương $\sum \dfrac{2a(ab+bc+ca)}{(b+c)(a+b+c)^2}\ge 1$
tương đương $\sum \dfrac{a}{b+c}\ge \dfrac{(a+b+c)^2}{2(ab+bc+ca)} $
(đugs theo svac)



#231364 Giúp em với

Đã gửi bởi tuan101293 on 10-03-2010 - 09:41 trong Phương trình hàm

f(x) ko cần liên tục đâu em,bài này chỉ đổi biến đơn thuần thôi :Rightarrow
Thay x=1 suy ra f(1)=3
Đặt $\dfrac{1}{x-1}=t$
khi đó ta được $f(1+\dfrac{1}{t-1})=2f(1+\dfrac{1}{t})-3$
đặt $g(x)=f(1+\dfrac{1}{x})-3$
suy ra $g(t-1)=2g(t)$ đặt $g(t)=l(t)*2^{-t}$ suy ra
l(t-1)=l(t).Đến đây em chọn hàm tuần hoàn tùy í có chu kì 1 là ok
:Rightarrow



#230662 Giúp em với

Đã gửi bởi tuan101293 on 03-03-2010 - 07:57 trong Phương trình hàm

Ví dụ bài này :
f:R vào R
$f(\dfrac{1}{2-x})=2f(x)-3$



#253436 bd thức

Đã gửi bởi tuan101293 on 19-02-2011 - 17:41 trong Bất đẳng thức - Cực trị

một bài nữa nè =D>
chưng minh với mọi a, b, c không âm ta có
$ \sum\dfrac{a(b+c)}{b^{2}+bc+c^{2}} \geq 2$

Bài này thì hay nhưng mà cũ lắm rồi
ý tưởng chỉ là côsi +svac (hack!!! )
$(b^2+bc+c^2)(ab+bc+ca)\le \dfrac{(b+c)^2(a+b+c)^2}{4}$



#253512 bd thức

Đã gửi bởi tuan101293 on 20-02-2011 - 13:48 trong Bất đẳng thức - Cực trị

A ơi! sử dụng bất đẳng thức đầu thì em dùng được rồi, nhưng chứng minh nó thế nào vậy? =D>
à mà nhân tiện anh với các bạn chém giúp em mấy bài này nữa( giải chi tiết nhé) cảm ơn nhiều nha!
1. cho các số thực dương a,b,c $\in [-1,1]$ thỏa mãn ab+bc+ca=1.
CmR:
$ \sum \dfrac{a^{2}+b^{2}}{(1-a^{2})(1-b^{2})} \geq \dfrac{9}{2} $
2. với a,b,c không âm. CmR:
$\sum \dfrac{1}{a} \geq \sum \dfrac{a+b}{c^{2}+ab}$
3. cho các số thực$ a,b,c \geq \dfrac{-3}{4}$ và a+b+c=1. CmR:
$\sum \dfrac{a}{1+a^{2}} \leq \dfrac{9}{10}$
4.cho a,b,c dương thỏa mãn a+b+c+1=4abc. CmR:
$ab+bc+ca \geq a+b+c$
5. cho a,b,c là độ dài ba cạnh của tam giác. CmR:
$4a^{2}b^{2}c^{2}\qeq (a+b-c)(b+c-a)(c+a-b)(a^{3}+b^{3}+c^{3}+abc)$

**************
bài đó thì thế này
theo côsi ta có $4(b^2+bc+c^2)(ab+bc+ca)\le (ab+bc+ca+b^2+bc+c^2)^2=((b+c)(a+b+c))^2$ (phân tích )
**************
chém luôn mấy bài sau nhé
1,dễ dàng CM được a,b,c cùng dấu (nếu ko sẽ có 1 số chui ra ngoài khoảng [-1,1])
ta có $(1-a^2)(1-b^2)\ge (1-ab)^2 (cauchy)=(bc+ca)^2=c^2(a+b)^2$
nên $VT\ge \sum \dfrac{(a^2+b^2)}{c^2(a+b)^2}\ge \sum \dfrac{1}{2c^2}\ge \sum \dfrac{1}{2ab}\ge \dfrac{9}{2(ab+bc+ca)}=\dfrac{9}{2}$
ĐPCM
2,bdt tương đương
$\sum \dfrac{(c-a)(c-b)}{c^3+abc}\ge 0$
giả sử $a\ge b\ge c$
suy ra $\dfrac{1}{c^3+abc}\ge \dfrac{1}{b^3+abc}$
sử dụng tiêu chuẩn vorni schur ta có ngay đpcm
3,đặt $f(x)=\dfrac{a}{1+a^2}$ có $f'(x)=\dfrac{1-a^2}{(1+a^2)^2}$
TH1: nếu có 2 số âm,1 số dương ,giả sử c>0;a,b<0 suy ra c>1 nên $A\le \dfrac{1}{2}+0+0<\dfrac{9}{10} $
TH2: nếu có 2 dương, 1 số âm thì tương tự
TH3: nếu cả 3 số cùng dương thì
$VT\le \sum\dfrac{a}{\dfrac{8}{9}+\dfrac{2a}{3}}\le \sum \dfrac{9a}{6a+8}=A$
ta sẽ Cm$A\le \dfrac{9}{10}$
tương đương $\sum \dfrac{1}{3a+2}\ge 1$ (đúng theo svac)
4,đặt $a=\dfrac{1}{2x},......$ thì $xy+yz+zx+2xyz=1$
nên tồn tại a,b,c(khác a,b,c ban đầu )mà $x=\dfrac{a}{b+c},y=\dfrac{b}{c+a},z=\dfrac{c}{a+b}$
nên ta cần CM $\sum \dfrac{(c+a)(c+b)}{2b*2c}\ge \sum \dfrac{a+b}{2c}$
tức là $\sum a(a+b)(a+c)\ge \sum 2ab(a+b)$ (đúng theo schur)
5,bdt tương đương
$\dfrac{abc}{\prod (a+b-c)}\ge \dfrac{a^3+b^3+c^3+abc}{4abc}$
tương đương (trử 2 vế cho 1)
$\dfrac{\sum a(a-b)(a-c)}{\prod (a+b-c)}\ge \dfrac{(a+b+c)(\sum (a-b)(a-c))}{4abc}$
tương đương
$\sum (a-b)(a-c)(\dfrac{a}{\prod (a+b-c)}-\dfrac{\sum a}{4abc})$
giả sử $a\ge b\ge c$
ta thấy bdt đúng theo vorni schur
đpcm



#220435 1/a^2+2bc

Đã gửi bởi tuan101293 on 15-11-2009 - 17:25 trong Bất đẳng thức - Cực trị

mình giải qua thế này nhé
Ta xét TH a,b,c>0
chú ý là
$\sum_{cyc}\dfrac{1}{a^2+2bc}=\dfrac{(\sum ab)(2\sum a^2+\sum ab)}{\prod(a^2+2bc)}$
tức là ta sẽ CM
$(2\sum a^2+\sum ab)(\sum a^2b^2)\ge \prod(a^2+2bc)$
(mình định thử côsi nhưng mà phải ăn cơm nên làm vội phân tích kiểu SOS và schur)
expand ra ta phải CM
$\sum_{cyc}2a^4(b-c)^2-\sum a^2b^2(c-b)(c-a)\ge 0$
đúng theo côsi 2 số
ĐPCM



#230448 Giúp em với

Đã gửi bởi tuan101293 on 28-02-2010 - 17:17 trong Phương trình hàm

E ko hiểu lắm, tại sao đến chỗ
$\dfrac{f(x)}{x^2}+1=\dfrac{(x+1)^2-f(x+1)}{x^2}$
lại suy ra f(x)=x với mọi x khác 0 nhỉ?
E hỏi thêm bài này luôn nhé:
Tìm hàm $ f: R \to R$ thỏa mãn $ f(3x-1)=f(2x)+x-1 \forall x \in R$
Em mới học pt hàm, thấy nó khó hiểu, mong anh thông cảm.

Cái đoạn trên thì em chú ý là f(x+1)=f(x),em thay vào giải là ok
Bài sau thì làm thế này :
Đặt g(x)=f(x)-x thì
g(3x-1)=g(2x)
Đặt g(x)=l(x-2)
suy ra l(3x)=l(2x)
suy ra $l(x)=l(\dfrac{3x}{2})$
đây là hàm nhân tính tuần hoàn chu kì 3/2.
đáp số f(x)=x+l(x-2) trong đó l(x) là hàm nhân tính tùy ý tuần hoàn chu kì 3/2.



#239173 Nice but maybe not very hard

Đã gửi bởi tuan101293 on 02-09-2010 - 20:13 trong Bất đẳng thức - Cực trị

Bài Toán :
Cho $4$ số thực dương $ a ; b ; c ; d $ thỏa mãn : $ abcd = 1$ ;

Chứng minh rằng ta có bất đẳng thức :

$ \dfrac{1}{(1+a)(1+a^2)} + \dfrac{1}{(1+b)(1+b^2)} + \dfrac{1}{(1+c)(1+c^2)} + \dfrac{1}{(1+d)(1+d^2)} \ge 1$
Ai đưa ra lời giải đẹp sớm nhất cho bài này ; thưởng 2$ ; Mại dzô ; mại dzô :)

Nguyễn Kim Anh

Bài này khó kinh,mình làm thử cách này(chưa check kỹ lắm,ai đó check hộ mình với)=))
đặt $f(x)=\dfrac{1}{(1+x)(1+x^2)}$ thì
**************
Bổ đề 1: nếu $ab\ge 1$

$f(a,b)\ge f(\sqrt{ab},\sqrt{ab})$ (CM=cách quy đồng trực tiếp)
và $f(a,b,c,d)\ge f(\sqrt[3]{abc},\sqrt[3]{abc},\sqrt[3]{abc})$ (CM như CM tử bdt cosi 2 số lên 3 số)
Bổ đề 2: nếu $ab\ge 1$ mà $a\ge 1$ và $b\le 1$ thì

$f(a,b)\ge f(ab,1)$ (CM=cách quy đồng)
**************
xét 3 trường hợp:
*****
TH1: 3 số nhỏ hơn 1,giả sử $b,c,d\le 1 $ suy ra $ab,ac,ad\ge 1$
ta có $f(a,b,c,d)\ge f(ab,1,c,d)\ge f(abc,d,1,1)=f(\dfrac{1}{d},d,1,1)\ge 1$
*****
TH2: 2 số nhỏ hơn 1, 2 số lớn hơn 1:
Giả sử $a\ge b\ge 1$ và $c,d\le 1$
Dễ thấy tồn tại ít nhất 1 trong 2 số ac,ad mà số này<1,giả sử $ac\ge 1$
ta có $f(a,b,c,d)\ge f(ac,b,d,1)\ge f(\sqrt{abc},\sqrt{abc},d,1)=f(x,x,\dfrac{1}{x^2},1)\ge 1 $
****
TH3: 1 số nhỏ hơn 1:
Giả sử $d\le 1$
ta có $f(a,b,c,d)\ge f(\sqrt[3]{abc},\sqrt[3]{abc},\sqrt[3]{abc},d)=f(x,x,x,\dfrac{1}{x^3})\ge 1$

ĐPCM



#239118 Nice but maybe not very hard

Đã gửi bởi tuan101293 on 02-09-2010 - 15:26 trong Bất đẳng thức - Cực trị

BDT cho n biến cũng đung đấy anh ak
$ \sum \limits_{i=1}^{n} \dfrac{1}{(a_i+1)(a_{i}^2+1)} \ge \dfrac{n}{4}$
Ta sẽ cm BDT sau: $\dfrac{1}{(1+a)(1+a^2)} \ge \dfrac{1}{4}-\dfrac{3}{8}ln a$
Thât vậy $f(x)=\dfrac{1}{(1+x)(1+x^2)} -\dfrac{1}{4}+\dfrac{3}{8}ln x$ :)
thì $f'(x)=\dfrac{(x-1)(3x^6+9x^5+18x^4+30x^3+39x^2+21x+8)}{8(1+x)^2(1+x^2)^2}$
i)$x \ge 1 \to f(x) \ge f(1)=0$
ii)$0<x<1 \to f(x) \ge f(1)=0$
Tóm lại :) đúng
Cộng từng vế có QED

PS: 2$ ???? :) :) USD hôm nay 19,5

Đạo hàm sai rồi em zai.
$f'(x)=\dfrac{(x-1)(3x^5+9x^4+18x^3+6x^2-x-3)}{8(1+x+x^2+x^3)^2x}$



#239120 Nice but maybe not very hard

Đã gửi bởi tuan101293 on 02-09-2010 - 15:32 trong Bất đẳng thức - Cực trị

khi đó $M\geq\dfrac{\displaystyle 1}{\displaystyle 2}[\dfrac{\displaystyle y^3}{\displaystyle x^3+y^3}+\dfrac{\displaystyle z^3}{\displaystyle y^3+z^3}+\dfrac{\displaystyle t^3}{\displaystyle z^3+t^3}+\dfrac{\displaystyle x^3}{\displaystyle t^3+x^3}]$
đến đây chỉ việc áp dụng bất đẳng thức Nesbitt cho 4 biến $x^3,y^3,z^3,t^3$ ta co ngay $M\geq1$

đây ko phải bdt nesbitt anh ơi.



#242608 Chém gió đê ( Anh em vào cho sôi động tí)

Đã gửi bởi tuan101293 on 01-10-2010 - 21:47 trong Dãy số - Giới hạn

Mình chém thử :(
(bực quá,bạn chọn cái alpha gõ mãi toàn lỗi,mình chuyển thành a cho nó ngon)
ta có
${x_n}^a={x_{n-1}}^a+x_{n-1}$
suy ra dãy tăng
suy ra $lim (x_1+...+x_n)= +\infty $
sử dụng ĐL stolz,ta quy về việc tìm
$S=lim (x_{n+1}-x_{n})=lim ((x_1+x_2+....+x_n)^\dfrac{1}{a}-(x_1+x_2+....+x_{n-1})^\dfrac{1}{a})$
đặt $(x_1+....+x_{n-1})^\dfrac{1}{a}=T$ thì $lim T=+ \infty $
và $0<S=(T^a+T)^\dfrac{1}{a}-T=T((1+\dfrac{1}{T^{a-1}})^\dfrac{1}{a}-1)<T(1+\dfrac{1}{a}*\dfrac{1}{T^{a-1}}-1)=\dfrac{1}{a*T^{a-2}}--->0$
suy ra lim =0